EXAM FINAL ECON (Chapter 10 quiz -> Chapter 15 quiz B)

Réussis tes devoirs et examens dès maintenant avec Quizwiz!

If the Fed wants to encourage nonbank lending, it will a) increase the IORB. b) decrease the ON RRP. c) decrease the IORB. d) increase the ON RRP.

B

If the effective federal funds rate is 1.64 percent, which of the following is most likely to be the Fed's target range for the federal funds rate? a) 1.00 to 2.00 b) 1.50 to 1.75 c) 1.14 to 2.14 d) 1.64 to 1.89

B

In terms of the mechanics of quantitative easing, a) quantitative easing formally changes interest rates; open-market operations only influence rates. b) it works the same as open-market operations. c) it differs from open-market operations in that the securities purchases occur directly from households. d) it only changes the interest rate; it doesn't influence bank reserves.

B

In the U.S. economy, the money supply is controlled by the a) U.S. Treasury. b) Federal Reserve System. c) Senate Committee on Banking and Finance. d) Congress.

B

Interest paid on reserve balances held at the Fed a) is available to the general public, but not to banks. b) will incentivize banks to hold more reserves and reduce riskier lending. c) is determined by the policy rate. d) amounts to just a few billion dollars a year.

B

Money in the U.S. is essentially debt of a) businesses and the banks. b) the Federal Reserve System and the banks. c) the national and local governments. d) businesses and the Federal Reserve System.

B

Open-market operations include a) the Fed communicating how it sees the current state of the economy. b) quantitative easing. c) paying interest on reserve balances held at Federal Reserve Banks. d) changes in the discount rate.

B

Open-market operations refer to a) purchases of stocks in the New York Stock Exchange. b) the purchases and sales of U.S. government securities by the Fed. c) central bank lending to banks. d) specifying the maximum loan amount for stock purchases.

B

Quantitative easing refers to a) the Fed buying short-term bonds to reduce short-term interest rates. b) the Fed buying longer-term bonds to reduce longer-term interest rates. c) the Fed selling short-term bonds to increase short-term interest rates. d) the Fed selling longer-term bonds to increase longer-term interest rates.

B

The Board of Governors of the Federal Reserve has _________ members. a) 5 b) 7 c) 9 d) 14

B

The Fed's response to the zero lower bound problem was a) to raise the lower bound. b) quantitative easing. c) quantitative tightening. d) restrictive monetary policy.

B

The Federal Reserve System a) has the same status as the Supreme Court. b) is basically an independent agency. c) has the status of a congressional committee. d) is an agency of the executive branch of the federal government.

B

The administered rates are the rates of interest at which a) Federal Reserve Banks lend to large corporations. b) set by a central bank to help it manage market-determined interest rates. c) that are not set by the Fed but influenced through buying and selling securities. d) banks lend to large corporations.

B

The discount rate is the interest a) rate at which the central banks lend to the U.S. Treasury. b) rate at which the Federal Reserve Banks lend to banks. c) yield on long-term government bonds. d) rate at which banks lend to the public.

B

The purpose of a restrictive monetary policy is to a) alleviate recessions. b) increase interest rates to rein in spending. c) increase aggregate demand and GDP. d) increase investment spending.

B

The sale of government securities by the Fed will cause a) bank reserves to increase. b) the money supply to decrease. c) demand deposits to increase. d) the interest rate to decrease.

B

The term "bankers' banks" means that the Federal Reserve a) will allow any employee of a commercial bank or thrift to open a checking account and obtain a loan at the central bank in their district. b) performs essentially the same functions for banks and thrifts as those institutions perform for the public. c) will allow the commercial banks and thrifts to open a reserve account with them, but will not lend them money. d) will allow the commercial banks and thrifts to borrow money from them (the Fed), but will not hold reserves in an account for them.

B

To say "money is what money does" means that a) money has been defined in a Constitutional amendment. b) whatever performs the functions of money extremely well is considered to be money. c) the money supply includes all public and private securities purchased by society. d) society, acting through Congress, specifies what shall be included in the money supply.

B

U.S. monetary policy is conducted by a) the president and Congress. b) the central bank. c) each individual state. d) the banks and thrifts.

B

When a consumer wants to compare the price of one product with another, money is primarily functioning as a a) store of value. b) unit of account. c) checkable deposit. d) medium of exchange.

B

The immediate short-run lasts a) exactly 4 weeks. b) as long as both input prices and output prices stay fixed. c) for one week or less. d) as long as input prices but not output prices stay fixed.

B Depending on the type of firm, the immediate short-run can last anywhere from a few days to a few months. It lasts as long as both input prices and output prices stay fixed.

A checking account balance is money because it a) is insured by the Federal Deposit Insurance Corporation. b) has been declared as such by the federal government. c) performs the functions of money. d) can be sold for currency.

C

After the mortgage default crisis and by the close of 2015, monetary policy consisted of a) stopping ZIRP and QE while starting QT. b) raising interest rates using forward guidance. c) both A and B. d) neither A nor B.

C

If the Federal Reserve System buys government securities, a) bank reserves will decline. b) interest rates on the securities will rise. c) the money supply will increase. d) the money supply will not change.

C

If the inflation rate is 2 percent and the unemployment rate is 6 percent, the Fed would likely choose a) a neutral monetary policy. b) a tight money policy. c) to lower the administered rates to increase bank lending. d) to raise interest rates.

C

If the inflation rate is at its target rate of 2 percent and the unemployment rate is close to or at the target rate of 3.5 percent, the Fed would likely choose a) an easy money policy. b) a tight money policy. c) a neutral monetary policy. d) to decrease the money supply with no change in interest rates.

C

If you are estimating your total expenses for school next semester, you are using money primarily as a) a medium of exchange. b) a store of value. c) a unit of account. d) an economic investment.

C

In a reverse repo transaction, a) banks return foreclosed property to previous owners. b) banks sell foreclosed property to new owners. c) the Fed borrows money from nonbank financial firms. d) the Fed loans money to nonbank financial firms.

C

In defining money as M1, economists exclude time deposits because a) the intrinsic value of time deposits is nil. b) the purchasing power of time deposits is much less stable than that of checkable deposits and currency. c) they are not directly or immediately a medium of exchange. d) they are not recognized by the federal government as legal tender.

C

In the United States, monetary policy is the responsibility of the a) U.S. Treasury. b) Department of Commerce. c) Federal Reserve System. d) U.S. Congress.

C

One major advantage of money serving as a medium of exchange is that it allows society to a) transfer purchasing power from the present to the future. b) measure the relative worth of products. c) escape the complications of barter. d) use credit cards instead of currency.

C

Purchasing groceries using a debit card best exemplifies money serving as a a) store of value. b) unit of account. c) medium of exchange. d) index of satisfaction.

C

Recalling the dual mandate bullseye chart, if an economy is in the northwest quadrant, the Fed should definitely a) loosen monetary policy. b) adopt a neutral monetary policy. c) use restrictive monetary policy. d) use expansionary monetary policy.

C

Small-denominated time deposits, by definition a) mature in one month or less. b) mature in one year or less. c) are less than $100,000. d) are held by state and local banks only.

C

Stabilizing a nation's price level and the purchasing power of its money can be achieved a) only with fiscal policy. b) only with monetary policy. c) with both fiscal and monetary policy. d) with neither fiscal nor monetary policy.

C

A decrease in the money supply a) increases the interest rate and decreases aggregate demand. b) increases both the interest rate and aggregate demand. c) lowers the interest rate and increases aggregate demand. d) lowers both the interest rate and aggregate demand.

A

A neutral monetary policy is a Fed policy in which a) the money supply and interest rates are left as they are. b) the money supply is increased but interest rates are not changed. c) both the money supply and interest rates are increased. d) the money supply is decreased and interest rates are increased.

A

An interest rate set by a central bank to help it manage market-determined interest rates defines a) an administered rate. b) the dollar/yen exchange rate. c) the Taylor rule. d) the policy rate.

A

As disposable income goes up, the a) average propensity to consume falls. b) average propensity to save falls. c) volume of consumption declines absolutely. d) volume of investment diminishes.

A

As it relates to Federal Reserve activities, the acronym FOMC describes the a) Federal Open Market Committee. b) Federal Options Market Committee. c) Federal Organization for Monetary Control. d) Federal Organization for Money Creation.

A

Checkable deposits are a) debts of commercial banks and savings institutions. b) debts of the federal government and government agencies. c) assets of the federal government and government agencies. d) assets of commercial banks and savings institutions.

A

How many members can serve on the Board of Governors of the Federal Reserve System? a) 7 b) 9 c) 12 d) 14

A

If the Fed wants to discourage bank lending, it will a) increase the interest paid on reserve balances held at the Fed. b) decrease the interest paid on reserve balances held at the Fed. c) buy government securities from banks. d) lower the policy rate.

A

If the Federal Reserve authorities were attempting to reduce inflation, the proper policies would be to a) Decrease the money supply with negative forward guidance, a higher federal funds target range, increases in the IORB and ON RRP, and/or quantitative tightening. b) Increase the money supply with positive forward guidance, a lower federal funds target range, reductions in the IORB and ON RRP, and/or quantitative easing. c) Increase the money supply by providing positive forward guidance and then increase the three administered rates. d) Buy bonds in the open market, lower the three administered rates, use ZIRP and QE.

A

If the economy appears to be operating at potential output, with stable prices and a low level of unemployment, the Fed would be inclined to choose a) a neutral monetary policy. b) an expansionary monetary policy. c) a restrictive monetary policy. d) a tight money policy.

A

If you write a check on a bank to purchase a used Honda Civic, you are using money primarily as a) a medium of exchange. b) a store of value. c) a unit of account. d) an economic investment.

A

In the United States, the money supply (M1) includes a) coins, paper money, checkable deposits, and savings deposits. b) currency, checkable deposits, and Series E bonds. c) coins, paper money, checkable deposits, and credit balances with brokers. d) paper money, coins, gold certificates, and time deposits.

A

The Federal Open Market Committee (FOMC) a) provides advice on banking stability to the Fed. b) monitors regulatory banking laws for member banks. c) sets policy on the sale and purchase of government bonds by the Fed. d) follows the actions and operations of financial markets to keep them open and competitive.

C

The Federal Reserve System changes the money supply by a) controlling the production of coins at the U.S. Mint. b) changing the number of banks in the system. c) providing forward guidance about how it intends to conduct monetary policy. d) restricting the issuance of Federal Reserve Notes because paper money is the largest portion of the money supply.

C

The Federal Reserve System's three administered rates are the a) interest on reserve balances rate, overnight reverse repo rate, and policy rate. b) federal funds rate, policy rate, and discount rate. c) IORB rate, ON RRP rate, and discount rate. d) inflation rate, unemployment rate, and economic growth rate.

C

The federal backing for money in the United States comes from a) providing sufficient quantities of precious metals, such as gold and silver, to cover the amount of paper money in circulation. b) pledging physical assets, such as land, natural resources, and public buildings, as collateral for outstanding currency. c) controlling the money supply in order to keep the value of money relatively stable over time. d) protecting checkable deposits at financial institutions with deposit guarantees.

C

The interest rate at which the Federal Reserve Banks lend to banks is called the a) policy rate. b) short-term rate. c) discount rate. d) federal funds rate.

C

Reserve balances are so-called because a) they are the funds that the Federal Reserve keeps on deposit for banks and thrifts to take care of ongoing commercial bank and thrift operations. b) banks and thrifts must call the Fed and request they (the Fed) "reserve" a certain amount of money each day for them (the commercial banks and thrifts) to be able to cover their transactions. c) the Fed asks banks and thrifts to "reserve" any money lent out as loans in a special account called "reserve balances" until the end of the week when it can then be released. d) none of the above.

A

The Fed's inability to stimulate the economy by reducing interest rates is known as the a) zero lower bound problem. b) zero upper bound problem. c) negative interest rate problem. d) quantitative easing problem.

A

The discount rate is the rate of interest at which a) Federal Reserve Banks lend to banks. b) savings and loan associations lend to some builders. c) Federal Reserve Banks lend to large corporations. d) banks lend to large corporations.

A

The federal funds rate is the interest rate that _________ charge(s) _________. a) banks; other banks b) the Fed; banks c) banks; their best corporate customers d) banks; on federal student loans

A

The seven members of the Board of Governors of the Federal Reserve System are a) appointed by the president with the confirmation of the Senate. b) elected by Congress from a slate of nominees provided by the president. c) appointed by the Senate Finance Committee. d) appointed by the presidents of the 12 Federal Reserve Banks.

A

To say that the Federal Reserve Banks are quasi-public banks means that a) they are privately owned but managed in the public interest. b) they deal only with banks of foreign nations and do not have direct business contact with U.S. banks. c) they deal only with commercial banks, and not the public. d) they are publicly owned but privately managed.

A

Which of the following Fed actions will decrease the money supply? a) raising the overnight reverse repo rate b) optimistic forward guidance c) open-market purchases of bonds d) raising taxes

A

Which of the following is paid to the Fed as a source of low-cost liquidity? a) discount rate b) interest on reserve balances c) policy rate d) overnight reverse repo rate

A

"Near-monies" are included in a) both M1 and M2. b) M2 only. c) M1 only. d) neither M1 nor M2.

B

Actions or communications by a central bank intended to help it achieve its macroeconomic policy objectives defines a) fiscal policy. b) monetary policy. c) trade policy. d) economic policy.

B

An increase in the money supply will a) lower interest rates and lower the equilibrium GDP. b) lower interest rates and increase the equilibrium GDP. c) increase interest rates and increase the equilibrium GDP. d) increase interest rates and lower the equilibrium GDP.

B

Assume the economy is operating at less than full employment. An expansionary monetary policy will cause interest rates to _________, which will _________ investment spending. a) decrease; decrease b) decrease; increase c) increase; increase d) increase; decrease

B

The money market comprises short-term lending markets that include markets for a) 20-year home mortgages. b) 20-year Treasury bonds. c) commercial paper. d) U.S. savings bonds.

C

The three main tools of monetary policy are a) interest on reserve balances, open-market operations, and changing the federal funds rate. b) tax-rate changes, changes in government expenditures, and open-market operations. c) open-market operations, forward guidance, and changing the administered interest rates. d) changes in government expenditures, changing the discount rate, and forward guidance.

C

Which of the following actions by the Fed will most likely increase bank lending? a) raising the policy rate of interest b) increasing the administered rates c) reducing the interest paid on reserve balances held at the Fed d) selling bonds on the open market

C

Why doesn't the Fed want to drive nominal interest rates below zero in response to a financial crisis and recession? a) Negative nominal interest rates would stimulate borrowing and spending, increasing aggregate demand. b) Negative interest rates would stimulate so much lending that it would unfairly increase the market power of banks. c) Negative nominal interest rates would cause people to withdraw their money from banks, reducing the ability of banks to extend loans. d) The Fed would lose the ability to raise the interest rates above zero in the future.

C

An expansionary monetary policy is a Fed policy in which a) the administered rates may be raised to discourage bank and nonbank lending. b) forward guidance concerning raising the federal funds target range may be provided. c) quantitative tightening may be necessary to increase longer-term rates. d) the administered rates may be lowered to encourage bank and nonbank lending.

D

An important routine function of the Federal Reserve System is to a) supervise the liquidation of the assets of bankrupt state banks. b) help large commercial banks develop correspondent relationships with smaller commercial banks. c) advise commercial banks as to the most profitable ways of reinvesting profits. d) provide facilities by which commercial banks and thrift institutions may collect on checks

D

As it relates to the aggregate-expenditures model, a leakage is a) an addition of spending into the income-expenditure stream: any increment to consumption, investment, government purchases, or net exports. b) a withdrawal of spending through taxes, government purchases, and exports. c) when someone other than the intended recipient intercepts and cashes a government stimulus check. d) a withdrawal of potential spending from the income-expenditures stream via saving, tax payments, or imports.

D

Big Bucks Bank currently holds $20 million in reserve balances. If the Fed increases the rate of interest on reserve balances held at the Fed, we would expect Big Bucks Bank to a) use those reserves to increase its consumer lending. b) not change its lending activity because only nonbanks are eligible to receive interest paid on reserve balances. c) use a big portion of those reserves to lend into the money market. d) hold even more reserves in its reserve account at the Fed, thereby reducing the amount it is willing to lend.

D

Currency held by banks is part of a) both the M1 and M2 definitions of the money supply. b) the M2 definition of the money supply only. c) the M1 definition of the money supply only. d) neither the M1 nor the M2 definition of the money supply.

D

Currency in circulation is part of a) M1 only. b) M2 only. c) neither M1 nor M2. d) both M1 and M2.

D

If the United States wants to increase its net exports, it might take steps to a) increase its GDP. b) reduce existing tariffs and import quotas. c) appreciate the dollar compared to foreign currencies. d) depreciate the dollar compared to foreign currencies.

D

If the effective federal funds rate is 1.24 percent, which of the following is most likely to be the Fed's target range for the federal funds rate? a) 1.24 to 1.49 b) 1.00 to 2.00 c) 1.14 to 2.14 d) 1.10 to 1.35

D

If the price index rises from 100 to 110, the purchasing power value of the dollar a) will rise by one-eleventh b) may either rise or fall. c) will rise by 20 percent. d) will fall by one-eleventh

D

Joe deposits $200 in currency into his checking account at a bank. This deposit is treated as a) a subtraction of $200 from the money supply because the $200 in currency is no longer in circulation. b) an addition of $200 to the money supply because of the creation of a checkable deposit of $200. c) an addition of $200 to the money supply because the bank holds $200 in currency and the checking account has been increased by $200. d) no change in the money supply because the $200 decrease in currency has been converted to a $200 increase in checkable deposits.

D

Money functions as a) a store of value. b) a unit of account. c) a medium of exchange. d) a store of value, a unit of account, and a medium of exchange.

D

Research suggests that a) the more independent the central bank, the lower the average annual growth of real GDP. b) the more independent the central bank, the higher the average annual growth of real GDP. c) there is no relationship between the degree of independence of a country's central bank and the growth rate of its real GDP. d) the less independent the central bank, the higher the average annual rate of inflation.

D

The Federal Open Market Committee (FOMC) is made up of a) the chair of the Board of Governors along with the 12 presidents of the Federal Reserve Banks. b) the seven members of the Board of Governors along with the president of the New York Federal Reserve Bank. c) the seven members of the Board of Governors of the Federal Reserve System along with the three members of the Council of Economic Advisers. d) the seven members of the Board of Governors of the Federal Reserve System along with the president of the New York Federal Reserve Bank and four other Federal Reserve Bank presidents on a rotating basis.

D

The Federal Reserve System was created in a) 1926. b) 1946. c) 1895. d) 1913.

D

The M1 money supply is composed of a) all coins and paper money held by the general public and the banks. b) bank deposits of households and business firms. c) bank deposits and mutual funds. d) checkable deposits, currency in circulation, and savings accounts.

D

The M2 money supply includes a) stock certificates. b) currency in bank vaults. c) the cash value of life insurance policies. d) individual shares in money market mutual funds.

D

The aggregate-demand curve a) is upsloping because a higher price level is necessary to make production profitable as production costs rise. b) is downward-sloping because production costs decline as real output increases. c) shows the amount of expenditures required to induce the production of each possible level of real output. d) shows the amount of real output that will be purchased at each possible price level.

D

The collateral used for overnight reverse repos is (are) a) corporate securities. b) autos. c) homes. d) government securities.

D

The group that sets the Federal Reserve System's policy on buying and selling government securities (bills, notes, and bonds) is the a) Federal Deposit Insurance Corporation (FDIC). b) Federal Bond Sale Authority. c) Council of Economic Advisers. d) Federal Open Market Committee (FOMC).

D

The interest rate that banks charge one another on overnight loans of currency held on deposit at the Fed is called the a) administered rate. b) interest on reserve balances. c) overnight lending rate. d) federal funds rate.

D

The paper money used in the United States is a) National Bank Notes. b) Treasury Notes. c) United States Notes. d) Federal Reserve Notes.

D

Upon which of the following industries is a restrictive monetary policy likely to be most effective? a) furniture b) clothing c) food processing d) residential construction

D

Which of the following best describes the cause-effect chain of expansionary monetary policy? a) A decrease in the money supply will lower the interest rate, increase investment spending, and increase aggregate demand and real GDP. b) A decrease in the money supply will raise the interest rate, decrease investment spending, and decrease aggregate demand and real GDP. c) An increase in the money supply will raise the interest rate, decrease investment spending, and decrease aggregate demand and real GDP. d) An increase in the money supply will lower the interest rate, increase investment spending, and increase aggregate demand and real GDP.

D

Which of the following is not a tool of monetary policy? a) quantitative tightening b) open market operations c) the IORB rate d) the Taylor rule

D

Which of the following is not part of the M2 money supply? a) money market mutual fund balances b) small-denominated time deposits c) currency d) large-denominated time deposits

D

Which of the following will happen when the Federal Reserve lowers the interest rate paid on reserve balances? a) Nonbank firms will choose to lend less money to the Fed. b) Nonbank firms will choose to lend more money to the Fed. c) Banks will choose to leave more of their money at the Fed overnight. d) Banks will choose to lend into the money market instead of lending to the Fed.

D

Which of the following would most likely result from the Fed imposing negative nominal interest rates in response to a financial crisis and recession? a) The negative interest rates would stimulate massive borrowing and spending, triggering rapid inflation in the short term. b) It would signal trouble to financial markets, causing people to deposit more money in banks to enhance feelings of financial security. c) Banks would freeze customer accounts so that they couldn't withdraw money, inciting financial panic. d) Bank customers would withdraw all their money from their accounts, banks would have less money to lend, and economic activity would fall.

D

Net exports are a) exports minus imports. b) imports minus exports. c) exports plus imports. d) always some positive amount.

A

When economists say that money serves as a store of value, they mean that it is a) a way to keep wealth in a readily spendable form for future use. b) a means of payment. c) a monetary unit for measuring and comparing the relative values of goods. d) declared as legal tender by the government.

A

When current tax revenues exceed current government expenditures and the economy is achieving full employment, a) the cyclically-adjusted budget has neither a deficit nor a surplus. b) the cyclically-adjusted budget may have either a deficit or a surplus. c) the cyclically-adjusted budget has a surplus. d) the government is engaging in an expansionary fiscal policy.

C

When economists say that money serves as a unit of account, they mean that it is a) a way to keep wealth in a readily spendable form for future use. b) a means of payment. c) a monetary unit for measuring and comparing the relative values of goods. d) declared as legal tender by the government.

C

An appropriate fiscal policy for severe demand-pull inflation is a) an increase in government spending. b) depreciation of the dollar. c) a reduction in interest rates. d) a tax rate increase.

D

If an unintended increase in business inventories occurs at some level of GDP, then GDP a) entails a rate of aggregate expenditures in excess of the rate of aggregate production. b) may be either above or below the equilibrium output. c) is too low for equilibrium. d) is too high for equilibrium.

D

The equilibrium price level and level of real output occur where a) real output is at its highest possible level. b) exports equal imports. c) the price level is at its lowest level. d) the aggregate demand and aggregate supply curves intersect.

D

The foreign purchases effect suggests that a decrease in the U.S. price level relative to other countries will a) shift the aggregate demand curve leftward. b) shift the aggregate supply curve leftward. c) decrease U.S. exports and increase U.S. imports. d) increase U.S. exports and decrease U.S. imports.

D

The purchasing power of the dollar a) has been increasing in recent years because of economic growth. Incorrect Response b) varies directly with the cost-of-living index. c) is inversely related to the level of aggregate demand. Correct Answer d) is the reciprocal of the price level.

D

Which of the following represents the most contractionary fiscal policy? a) a $30 billion tax cut b) a $30 billion increase in government spending c) a $30 billion tax increase d) a $30 billion decrease in government spending

D

A $80 price tag on a sweater in a department store window is an example of money functioning as a a) unit of account. b) standard of deferred payments. c) store of value. d) medium of exchange.

A

Actual investment equals saving a) at all levels of GDP. b) at all below-equilibrium levels of GDP. c) at all above-equilibrium levels of GDP. d) only at the equilibrium GDP.

A

An input whose price is often fixed in both the immediate-short-run and short-run is a) labor, due to labor contracts. b) gasoline, due to supply issues. c) both A and B. d) neither A nor B.

A

Discretionary fiscal policy will stabilize the economy most when a) deficits are incurred during recessions and surpluses during inflations. b) the budget is balanced each year. c) deficits are incurred during inflations and surpluses during recessions. d) budget surpluses are continuously incurred.

A

If consumers expect their future real income to rise, they will a) tend to spend more of their current incomes now. b) tend to spend less of their current incomes now. c) not change their current spending plans. d) tend to save more money now.

A

If the MPC is 0.8 and disposable income is $200, then a) consumption and saving cannot be determined from the information given. b) saving will be $20. c) personal consumption expenditures must be $160. d) saving will be $40.

A

In a mixed open economy, the equilibrium GDP is determined at that point where a) Sa + M + T = Ig + X + G. b) the 45-degree line and the saving schedule intersect. c) Sa + X + G = Ig + T. d) Sa + Ig + X = G + T.

A

Other things equal, an increase in nominal wages will a) shift the aggregate supply curve to the left. b) shift the aggregate supply curve to the right. c) shift the aggregate demand curve to the left. d) shift the aggregate demand curve to the right.

A

PayPal and Venmo are a) cash-transfer systems. b) federally chartered commercial banks. c) unchartered credit unions. d) mutual savings banks.

A

The aggregate-expenditures model is built upon which of the following assumptions? a) Prices are fixed. b) The economy is at full employment. c) Prices are fully flexible. d) Government spending policy has no ability to affect the level of output.

A

C = 40 + 0.8Y Ig = 40 X = 20 M = 30 (Advanced analysis) The equations give information for a private open economy. The letters Y, C, Ig, X, and M stand for GDP, consumption, gross investment, exports, and imports, respectively. Figures are in billions of dollars. The equilibrium GDP (=Y) in the economy is a) $200. b) $245. c) $320. d) $350.

D

Which of the following is a tool of monetary policy? a) open-market operations b) changes in banking laws c) changes in tax rates d) changes in government spending

A

Monetary policy is expected to have its greatest impact on a) Xn. b) Ig. c) C. d) G.

B

The functions of money are to serve as a a) resource allocator, method for accounting, and means of income distribution. b) unit of account, store of value, and medium of exchange. c) determinant of consumption, investment, and government spending. d) factor of production, exchange, and aggregate supply.

B

The interest on reserve balances is the interest a) rate banks pay the Fed to keep their money overnight. b) rate the Fed pays banks for any money they deposit on an overnight basis. c) rate at which banks lend to the public. d) rate at which the central banks lend to the U.S. Treasury.

B

The interest rate that the Fed uses as the policy rate is called the a) administered rate. b) federal funds rate. c) IORB rate. d) ON RRP rate.

B

The long-run begins a) before the immediate short-run ends. b) after the short-run ends. c) after the immediate short-run ends. d) before the short-run ends.

B

Which of the following is not a tool of monetary policy? a) open-market operations b) changes in banking laws c) changes in one or more of the three administered interest rates d) the Fed communicating to the public how it intends to manage monetary policy

B

(Consider This) Credit cards are a) the fastest-growing component of the M1 money supply. b) near-monies that are part of the M2 money supply but not the M1 money supply. c) not money, as officially defined. d) also known as time deposits.

C

Dissaving occurs when a) income exceeds consumption. b) saving exceeds consumption. c) consumption exceeds income. d) saving exceeds income.

C

The short-run begins a) before the immediate short-run ends. b) after the long run ends. c) after the immediate short run ends. d) before the long run ends.

C

A $20 bill is a a) gold certificate. b) Treasury Note. c) Treasury bill. d) Federal Reserve Note.

D

Imports have the same effect on the current size of GDP as a) exports. b) investment. c) consumption. d) saving.

D

The IORB rate is the interest rate that _________blank pay(s) _________blank. a) banks; the Fed b) banks; on federal student loans c) banks; their best corporate customers d) the Fed; banks

D

The foreign purchases effect a) shifts the aggregate demand curve rightward. b) shifts the aggregate demand curve leftward. c) shifts the aggregate supply curve rightward. d) moves the economy along a fixed aggregate demand curve.

D

If the price index rises from 100 to 180, then the purchasing power of the dollar will fall by about a) 20 percent. b) 100 percent. c) 80 percent. d) 22 percent. e) 44 percent.

E

The interest-rate effect suggests that a) a decrease in the supply of money will increase interest rates and reduce interest-sensitive consumption and investment spending. b) an increase in the price level will increase the demand for money, reduce interest rates, and decrease consumption and investment spending. c) an increase in the price level will increase the demand for money, increase interest rates, and decrease consumption and investment spending. d) an increase in the price level will decrease the demand for money, reduce interest rates, and increase consumption and investment spending.

C

Which of the following statements about the real balances effect is not correct? a) When the price level rises, real balances fall, purchasing power declines, and people demand less output. b) When the price level falls, real balances increase, purchasing power rises, and people demand more output. c) A higher price level means more consumption spending while a lower price level means less consumer spending. d) Real balances (the purchasing power of nominal balances) vary inversely with the price level.

C

In a private closed economy, when aggregate expenditures exceed GDP, a) GDP will decline. b) business inventories will rise. c) saving will decline. d) business inventories will fall.

D

The real-balances effect indicates that a) an increase in the price level will increase the demand for money, increase interest rates, and reduce consumption and investment spending. b) a lower price level will decrease the real value of many financial assets and therefore reduce spending. c) a higher price level will increase the real value of many financial assets and therefore increase spending. d) a higher price level will decrease the real value of many financial assets and therefore reduce spending.

D

When the economy is at full employment, a) one cannot generalize in comparing the actual and the cyclically-adjusted budgets. b) the cyclically-adjusted budget will show a surplus and the actual budget will show a deficit. c) the actual budget will show a surplus and the cyclically-adjusted budget will show a deficit. d) the actual and the cyclically-adjusted budgets will be equal.

D

Built-in stability means that a) an annually balanced budget will offset the procyclical tendencies created by state and local finance and thereby stabilize the economy. b) with given tax rates and expenditures policies, a rise in domestic income will reduce a budget deficit or produce a budget surplus, while a decline in income will result in a deficit or a lower budget surplus. c) Congress will automatically change the tax structure and expenditure programs to correct upswings and downswings in business activity. d) government expenditures and tax receipts automatically balance over the business cycle, though they may be out of balance in any single year.

B

Exports have the same effect on the current size of GDP as a) imports. b) investment. c) taxes. d) saving.

B

In an effort to avoid a recession, the government implements a tax rebate program, effectively cutting taxes for households. We would expect this to a) affect neither aggregate supply nor aggregate demand. b) increase aggregate demand. c) reduce aggregate demand. d) reduce aggregate supply.

B

Money market deposit accounts are included in a) M1 only. b) M2 only. c) neither M1 nor M2. d) both M1 and M2.

B

Stock market price quotations best exemplify money serving as a a) store of value. b) unit of account. c) medium of exchange. d) index of satisfaction.

B

The amount by which government expenditures exceed revenues during a particular year is the a) public debt. b) budget deficit. c) full employment. d) GDP gap.

B

When economists say that money serves as a medium of exchange, they mean that it is a) a way to keep wealth in a readily spendable form for future use. b) a means of payment. c) a monetary unit for measuring and comparing the relative values of goods. d) declared as legal tender by the government.

B

At equilibrium real GDP in a private closed economy, a) the MPC must equal the APC. b) the slope of the aggregate-expenditures schedule equals the MPS. c) aggregate-expenditures and real GDP are equal. d) planned saving and consumption are equal.

C

In a mixed closed economy, a) government purchases and saving are injections, while investment and taxes are leakages. b) taxes and government purchases are leakages, while investment and saving are injections. c) taxes and savings are leakages, while investment and government purchases are injections. d) taxes and investment are injections, while saving and government purchases are leakages.

C

In a private closed economy, when aggregate expenditures equal GDP, a) consumption equals investment. b) consumption equals aggregate expenditures. c) planned investment equals saving. d) disposable income equals consumption minus saving.

C

The 45-degree line on a graph relating consumption and income shows a) all the points where the MPC is constant. b) all the points at which saving and income are equal. c) all the points at which consumption and income are equal. d) the amounts households will plan to save at each possible level of income.

C

The federal budget deficit is found by a) subtracting government tax revenues plus government borrowing from government spending in a particular year. b) subtracting government tax revenues from government spending in a particular year. c) cumulating the differences between government spending and tax revenues over all years since the nation's founding. d) subtracting government revenues from the noninvestment-type government spending in a particular year.

B

The foreign purchases effect suggests that an increase in the U.S. price level relative to other countries will a) increase the amount of U.S. real output purchased. b) increase U.S. imports and decrease U.S. exports. c) increase both U.S. imports and U.S. exports. d) decrease both U.S. imports and U.S. exports.

B

The aggregate supply curve (short-run) is upsloping because a) wages and other resource prices match changes in the price level. b) the price level is flexible upward but inflexible downward. c) per-unit production costs rise as the economy moves toward and beyond its full-employment real output. d) wages and other resource prices are flexible upward but inflexible downward.

C

The cyclically-adjusted budget refers to a) the inflationary impact that the automatic stabilizers have in a full-employment economy. b) that portion of a full-employment GDP that is not consumed in the year it is produced. c) the size of the federal government's budgetary surplus or deficit when the economy is operating at full employment. d) the number of workers who are underemployed when the level of unemployment is 3 to 4 percent.

C

The cyclically-adjusted budget tells us a) that in a full-employment economy, the federal budget should be in balance. b) that tax revenues should vary inversely with GDP. c) what the size of the federal budget deficit or surplus would be if the economy was at full employment. d) the actual budget deficit or surplus realized in any given year.

C

The definition of a lump-sum tax is a) A tax that is levied only on imports, a lump-sum tax is never levied on exports. b) A tax that generates revenues all at one time and only once; it does not reoccur. c) A tax that collects a constant amount (the tax revenue of government is the same) at all levels of GDP. d) A tax that varies with GDP; the higher the GDP, the higher the tax.

C

The public debt is the amount of money that a) state and local governments owe to the federal government. b) Americans owe to foreigners. c) the federal government owes to holders of U.S. securities. d) the federal government owes to taxpayers.

C

The short-run aggregate supply curve has a) a constant slope. b) a negative slope. c) a slope that is not constant. d) the same slope as the long-run aggregate supply curve.

C

Which one of the following would increase per-unit production cost and therefore shift the aggregate supply curve to the left? a) a reduction in business taxes b) production bottlenecks occurring when producers near full plant capacity c) an increase in the price of imported resources d) deregulation of industry

C

Assume there are no prospective investment projects (I) that will yield an expected rate of return (r) of 25 percent or more, but there are $5 billion of investment opportunities with an expected rate of return between 20 and 25 percent, an additional $5 billion between 15 and 20 percent, and so on. If the real interest rate is 15 percent in this economy, the aggregate amount of investment will be a) $25 billion. b) $20 billion. c) $15 billion. d) $10 billion.

D

During periods of rapid inflation, money may cease to work as a medium of exchange a) unless it has been designated legal tender. b) unless it is backed by gold. c) because it is too scarce for everyone to have enough for transactions. d) because people and businesses will not want to accept it in transactions.

D

Higher interest rates may cause a) consumers to decide not to purchase a new house or new automobile. b) businesses to postpone a potential purchase of capital. c) an increase in the amount of investment spending. d) both A and B.

D

If Congress adjusted the U.S. tax system so that taxes were more progressive, the a) economy would become more inflation prone. b) economy would become less stable. c) stability of the economy would be unaffected. d) economy would become more stable.

D

If at some level of GDP the economy is experiencing an unintended decrease in inventories, a) the aggregate level of saving will decline. b) the price level will fall. c) the business sector will lay off workers. d) domestic output will increase.

D

If the multiplier in an economy is 5, a $10 billion increase in net exports will a) reduce GDP by $2 billion. b) decrease GDP by $50 billion. c) increase GDP by $10 billion. d) increase GDP by $50 billion.

D

Other things equal, an increase in an economy's exports will a) lower the marginal propensity to import. b) have no effect on domestic GDP because imports will change by an offsetting amount. c) decrease its domestic aggregate expenditures and therefore decrease its equilibrium GDP. d) increase its domestic aggregate expenditures and therefore increase its equilibrium GDP.

D

The APC can be defined as the fraction of a a) change in income that is not spent. b) change in income that is spent. c) specific level of total income that is not consumed. d) specific level of total income that is consumed.

D

The U.S. public debt a) refers to the debts of all units of government—federal, state, and local. b) consists of the total debt of U.S. households, businesses, and government. c) refers to the collective amount that U.S. citizens and businesses owe to foreigners. d) consists of the historical accumulation of all past federal deficits and surpluses.

D

If the MPC in an economy is 0.75, government could shift the aggregate demand curve leftward by $60 billion by a) reducing government expenditures by $12 billion. b) reducing government expenditures by $60 billion. c) increasing taxes by $15 billion. d) increasing taxes by $20 billion.

D Find the multiplier with the formula, 1 divided by 1 minus the MPC: 4 [1 ÷ (1 − 0.75)]. Divide the desired decrease of $60 billion by the multiplier to find how much government spending should decrease: $15 ($60 ÷ 4). Taxes would need to increase by more than that since some of the tax increase will reduce consumption spending and some will reduce saving: [($60 ÷ 4) = $15; $15 ÷ 0.75 = $ 20 billion.]

If the equilibrium level of GDP in a private open economy is $1,000 billion and consumption is $700 billion at that level of GDP, then a) saving must be $300 billion. b) net exports must be $300 billion. c) S + C must equal $300 billion. d) Ig + Xn must equal $300 billion.

D The formula to calculate this is GDP = C + Ig + Xn: so, if consumption is $700, then Ig + Xn is $300 billion: $1,000 = $700 + Ig + Xn.

What function is money serving when you deposit money in a savings account? a) a store of value b) a unit of account c) a checkable deposit d) a medium of exchange

A

Checkable deposits are a) included in M1. b) not included in either M1 or M2. c) considered to be one of the near-monies. d) also called time deposits.

A

Checkable deposits are classified as money because a) they can be readily used in purchasing goods and paying debts. b) banks hold currency equal to the value of their checkable deposits. c) they are ultimately the obligations of the Treasury. d) they earn interest income for the depositor.

A

If the inflation rate was on target at 2 percent and the unemployment rate was 3.4 percent, the Fed would likely adopt a(n) a) easy money policy. b) neutral monetary policy. c) expansionary monetary policy. d) restrictive monetary policy.

B

If the marginal propensity to consume is 0.7, then the marginal propensity to save must be a) 1.3. b) 0.3. c) 0.7. d) 1.0

B

In a reverse repo transaction, the Fed _________ money. a) loans b) borrows c) prints new d) donates

B

The immediate-short-run aggregate supply curve represents circumstances where a) both input and output prices are fixed. b) both input and output prices are flexible. c) input prices are fixed, but output prices are flexible. d) input prices are flexible, but output prices are fixed.

A

The money supply is backed a) by the government's ability to control the supply of money and therefore to keep its value relatively stable. b) by government bonds. c) dollar-for-dollar by gold and silver. d) by gold reserves representing a fraction of the total value of dollars in circulation.

A

The reason the long-run aggregate supply curve is vertical is a) when both input prices and output prices are flexible, profit levels always adjust to give firms exactly the right profit incentive to produce the full-employment output level. b) when input prices and output prices are fixed, output levels adjust to ensure profit always equals zero. c) that the total amount of output supplied in the economy depends directly on the volume of spending that results at a constant price level. d) none of the above.

A

What "backs" the money supply of the United States? a) the U.S. government's ability to keep the value of money relatively stable b) the amount of gold the U.S. government has on deposit at its banks c) the fact that currency is issued by the Federal Reserve System d) the fact that the intrinsic value of coins in circulation is greater than their face value

A

Which of the following statements about the Fed's dual mandate is correct? a) The Fed gives more weight to the full-employment target than the inflation target because very bit of cyclical unemployment represents personal misery for those left unemployed. b) The full-employment target is weighted more than the inflation target because the Fed does not wish to adversely affect the income tax revenues collected by the U.S. Treasury. c) The inflation rate target is weighted more than the full-employment target because of the many price-sensitive retired consumers in the country. d) The inflation rate target is weighted more than the full-employment target because of how quickly inflation can get out of hand compared to unemployment which changes slowly.

A

If the MPC in an economy is 0.80, government could shift the aggregate demand curve leftward by $32 billion by a) increasing taxes by $8 billion. b) increasing taxes by $6.4 billion. c) reducing government expenditures by $32 billion. d) reducing government expenditures by $4 billion.

A Find the multiplier using the formula, 1 ÷ 1 − MPC, so the Multiplier = 1 ÷ 1 − 0.80 = 5. Then begin a process of elimination by first using the multiplier to find how much government expenditures need to decrease to get a decrease in aggregate demand of $32; so, $32 ÷ 5 = $6.4. And then determine how much taxes need to increase to get a left shift in aggregate demand using the multiplier of 5. So, $32 ÷ 5 = $6.4. But households use their income to both consume and save, so divide that amount by the MPC, $6.4 ÷ 0.8. Then find your answer from the choices given.

Suppose the price level is fixed, the MPC is 0.50, and the GDP gap is a negative $84 billion. To achieve full-employment output (exactly), government should a) reduce taxes by $84 billion. b) increase government expenditures by $84 billion. c) reduce taxes by $42 billion. d) reduce government expenditures by $42 billion.

A Find the multiplier with the formula, 1 divided by 1 minus the MPC: 2 [1 ÷ (1 − 0.50)]. Divide the desired increase of $84 billion by the multiplier to find how much government spending should increase: $42 ($84 ÷ 2). Taxes would need to decrease by more than that since some of the tax decrease will be spent (the MPC is 0.50) and some will be saved (the MPS is 0.50): [($84 ÷ 2) = $42; $42 ÷ 0.50 = $84 billion]. Taxes should be reduced by $84 billion.

If a $25 billion increase in government expenditures increases equilibrium GDP by $100 billion, then a) the MPS is 0.25 for this economy. b) the MPC is 1.0 in this economy. c) none of the above. d) the multiplier is 2.5.

A The multiplier is 4 ($100 ÷ $25). Find the MPS with the formula [multiplier = 1 ÷ MPS] and solve for the MPS. The MPS is 0.25 [4 = 1 ÷ MPS].

If the MPS in an economy is 0.40, government could shift the aggregate demand curve leftward by $60 billion by a) reducing government expenditures by $24 billion. b) reducing government expenditures by $125 billion. c) increasing taxes by $250 billion. d) increasing taxes by $60 billion.

A Find the multiplier with the formula, 1 divided by the MPS: 2.5 (1 ÷ 0.40). Divide the desired decrease of $60 billion by the multiplier to find how much government spending should decrease: $24 ($60 ÷ 2.5). Taxes would need to increase by more than that since some of the tax increase will reduce consumption spending (1 = MPC + MPS so the MPC is 0.60) and some will reduce saving (the MPS is 0.40): [($60 ÷ 2.5) = $24; $24 ÷ 0.60 = $40.0 billion.]

If the multiplier in an economy is 5, a $20 billion increase in net exports will a) increase GDP by $100 billion. b) reduce GDP by $4 billion. c) decrease GDP by $100 billion. d) increase GDP by $20 billion.

A $20 × 5 = $100. The increase in net exports will increase GDP by $100 billion.

Which of the following represents the most expansionary fiscal policy? a) a $10 billion tax cut b) a $10 billion increase in government spending c) a $10 billion tax increase d) a $10 billion decrease in government spending

B

A withdrawal of potential spending from the income-expenditures stream via saving, tax payments, or imports is a(n) a) injection. b) leakage. c) expenditure gap. d) tariff.

B

An economy is employing 4 units of capital, 7 units of raw materials, and 10 units of labor to produce its total output of 670 units. Each unit of capital costs $12; each unit of raw materials, $6; and each unit of labor, $3. If the per-unit price of raw materials rises from $6 to $10 and all else remains constant, the per-unit cost of production will rise by about a) 52 percent. b) 10 percent. c) 42 percent. d) 100.1 percent.

B Initially, total input costs = (4 × $12) + (7 × $6) + (10 × $3) = $120. Divide total input cost by total output to find per-unit cost of production: $120 ÷ 670 = $0.2. With an increase in the price of raw materials, total input costs = (4 × $12) + (7 × $10) + (10 × $3) = $148. Divide total input cost by total output to find the new per-unit cost of production: $670 ÷ 148 = $0.22. The percentage change in per-unit costs are [($0.22 − $0.2) ÷ $0.2] = 0.1 × 100.1 = 10 percent.

If investment decreases by $21 billion and the economy's MPC is 0.5, the aggregate demand curve will shift a) leftward by $105 billion at each price level. b) leftward by $42 billion at each price level.. c) rightward by 42 billion at each price level.. d) rightward by $21 billion at each price level.

B The multiplier is 2 [1 ÷ (1 - MPC)]. Multiply the decrease in investment times the multiplier to find how much the AD curve will shift. It will shift leftward (a decrease) by $42 billion ($21 × 2).

A major advantage of the built-in or automatic stabilizers is that they a) simultaneously stabilize the economy and reduce the absolute size of the public debt. b) automatically produce surpluses during recessions and deficits during inflation. c) require no legislative action by Congress to be made effective. d) guarantee that the federal budget will be balanced over the course of the business cycle.

C

A private closed economy includes a) households, businesses, and government, but not international trade. b) households, businesses, and international trade, but not government. c) households and businesses, but not government or international trade. d) households only.

C

An appropriate fiscal policy for a severe recession is a) an increase in payroll taxes. b) a decrease in government spending. c) an increase in government spending. d) an increase in interest rates.

C

An effective expansionary fiscal policy will a) reduce a cyclical deficit but necessarily increase the actual deficit. b) reduce the cyclically-adjusted deficit. c) increase the cyclically-adjusted deficit but reduce the actual deficit. d) always result in a balanced actual budget once full employment is achieved.

C

Suppose the price level is fixed, the MPC is 0.5, and the GDP gap is a negative $80 billion. To achieve full-employment output (exactly), government should a) increase government expenditures by $80 billion. b) reduce government expenditures by $40 billion. c) reduce taxes by $40 billion. d) reduce taxes by $80 billion.

D Find the multiplier with the formula, 1 divided by 1 minus the MPC: 2 [1÷ (1 − 0.5)]. Divide the desired increase of $80 billion by the multiplier to find how much government spending should increase: $40 ($80 ÷ 2). Taxes would need to decrease by more than that since some of the tax decrease will be spent (the MPC is 0.5) and some will be saved (the MPS is 0.5): [($80 ÷ 2) = $40; $40 ÷ 0.5 = $80 billion]. Taxes should be reduced by $80 billion.

For a private closed economy, an unintended decline in inventories suggests that a) aggregate expenditures are less than the business sector expected them to be. b) aggregate expenditures exceed production. c) actual investment exceeds saving. d) planned investment is greater than consumption.

B

In a certain year, the aggregate amount demanded at the existing price level consists of $100 billion of consumption, $40 billion of investment, $10 billion of net exports, and $20 billion of government purchases. Full-employment GDP is $120 billion. To obtain price level stability under these conditions, the government should a) increase tax rates and/or reduce government spending. b) discourage personal saving by reducing the interest rate on government bonds. c) increase government expenditures. d) encourage private investment by reducing corporate income taxes.

A Aggregate demand = C + Ig + G + Xn: $100 + $40 + $20 + $10 = $170. There is an inflationary gap and the actions that should be taken are to increase taxes and decrease government spending.

The amount of real output that could be purchased at current prices if all of our assets were liquidated (turned to cash) and the money used to purchase goods and services is the a) real-balances effect. b) interest-rate effect. c) foreign-purchases effect. d) liquidation effect.

A An individual's real balances are equal to the amount of real output that she could purchase at current prices if she liquidated (turned to cash) all of her assets and used the money to purchase goods and services.

Suppose the government purposely changes the economy's cyclically-adjusted budget from a deficit of 0 percent of real GDP to a deficit of 3 percent of real GDP. The government is engaging in a(n) a) expansionary fiscal policy. b) contractionary fiscal policy. c) neutral fiscal policy. d) low-interest-rate policy.

A

The aggregate supply curve a) is explained by the interest-rate, real-balances, and foreign purchases effects. b) gets steeper as the economy moves from the top of the curve to the bottom of the curve. c) shows the various amounts of real output that businesses will produce at each price level. d) is downward-sloping because real purchasing power increases as the price level falls.

C

In contrast to investment, consumption is a) relatively unstable. b) relatively stable. c) measurable. d) unmeasurable.

B

The crowding-out effect of expansionary fiscal policy suggests that a) tax increases are paid primarily out of saving and therefore are not an effective fiscal device. b) increases in government spending financed through borrowing will increase the interest rate and thereby reduce investment. c) it is very difficult to have excessive aggregate spending in the U.S. economy. d) consumer and investment spending always vary inversely.

B

An aggregate-expenditures schedule is a a) curve or schedule that shows the amounts that firms plan to invest at various possible values of real gross domestic product (real GDP). b) table of numbers showing the total amount spent on final goods and final services at different levels of real gross domestic product (real GDP). c) table of numbers showing the total sales tax revenue generated on purchases of final goods and services. d) none of the above.

B

An increase in input productivity will a) shift the aggregate supply curve leftward. b) reduce the equilibrium price level, assuming downward flexible prices. c) reduce the equilibrium real output. d) reduce aggregate demand.

B

An investment-demand curve is a curve that shows the amounts of investment demanded by an economy at a series of a) nominal interest rates. b) real interest rates. c) real inflation rates. d) nominal saving rates.

B

An upward shift of the aggregate-expenditures schedule might be caused by a) a decrease in exports, with no change in imports. b) a decrease in imports, with no change in exports. c) an increase in exports, with an equal decrease in investment spending. d) an increase in imports, with no change in exports.

B

Assume that in a private closed economy, consumption is $240 billion and investment is $50 billion, both at the $280 billion level of domestic output. Thus, a) saving is $10 billion. b) unplanned decreases in inventories of $10 billion will occur. c) the MPC is 0.80. d) unplanned increases in inventories of $10 billion will occur.

B

The equilibrium level of GDP is associated with a) an excess of planned investment over saving. b) no unintended changes in inventories. c) an unintended decrease in business inventories. d) an unintended increase in business inventories.

B

Assume the economy is at full employment and that investment spending declines dramatically. If the goal is to restore full employment, government fiscal policy should be directed toward a) an equality of tax receipts and government expenditures. b) an excess of tax receipts over government expenditures. c) an excess of government expenditures over tax receipts. d) a reduction of subsidies and transfer payments and an increase in tax rates.

C

If the equilibrium level of GDP in a private open economy is $1,400 billion and consumption is $950 billion at that level of GDP, then a) saving must be $450 billion. b) S + C must equal $450 billion. c) Ig + Xn must equal $450 billion. d) net exports must be $450 billion.

C

Other things equal, an improvement in productivity will a) shift the aggregate demand curve to the left. b) shift the aggregate supply curve to the left. c) shift the aggregate supply curve to the right. d) increase the price level.

C


Ensembles d'études connexes

global humanities test 1 Which city is most associated with the High Renaissance in Italy?

View Set

CS 665- Household Risk Management Exam #2 (Chapter 11-12)

View Set

Adobe Dreamweaver Certification Study Guide

View Set